fmCASES Questions

¡Supera tus tareas y exámenes ahora con Quizwiz!

Which of the following is a comorbid condition that needs to be controlled in order to improve asthma symptoms? A. High BMI B. Insomnia C. Atopic dermatitis D. Anxiety E. Anemia

The correct answer is A. When a patient experiences difficulty with asthma control, the physician must consider and address comorbid conditions known to effect asthma control. These comorbid conditions include: gastroesophageal reflux (GERD), chronic sinusitis/uncontrolled allergic rhinitis, stress/depression, obstructive sleep apnea, and being overweight or obese. Successful treatment of these conditions often results in improved control of the patient's asthma symptoms.

A 9-month old baby boy comes to the clinic for a well-child visit. The child is at the 50th percentile for weight, length, and head circumference. He is reaching all developmental milestones appropriately. The mother has no concerns at this visit. The child has previously received the following vaccines: 3 doses of DTaP, 3 doses of Hib, 2 doses of HepB, 3 doses of RotaV, 2 doses of IPV and 3 doses of PCV13, and no influenza vaccines. Which vaccines should the child receive at today's visit? A. Influenza, Hep B, IPV, DTaP B. Influenza, IPV C. Influenza, Hep B, IPV D. Hep B, DTaP, IPV E. Hep B, IPV, and MMR

The correct answer ic C. A. Influenza, Hep B, IPV, DTaP is incorrect. All three doses of DTaP have been given. B. Influenza, IPV is incorrect. The patient needs the third Hep B shot. C. Influenza, Hep B, IPV is correct. The patient needs a third Hep B, a third IPV, and a yearly flu shot starting at 6 months of age. D. Hep B, DTaP, IPV is incorrect. All three doses of DTaP have been given and the patient now needs a yearly flu shot starting at 6 months of age. E. Hep B, IPV, and MMR is incorrect. The patient also needs a yearly flu shot starting at 6 months of age and MMR is not given before 12 months of age.

Mark is a 5-month-old male who is brought to the urgent care clinic with a three-day history of rhinorrhea and non-productive cough. When he was born he was large for gestational age, and his exam then was notable for macrocephaly, macroglossia, and hypospadias. On physical exam now his vitals signs are stable. He has copious nasal discharge, but his lungs are clear to auscultation. On abdominal exam, you palpate an abdominal mass on the right side just below the subcostal margin. It is 7 cm in diameter and does not cross the midline. The abdomen is soft and non-tender with active bowel sounds. What is the most likely cause of his mass? A. Wilms' tumor B. Teratoma C. Renal cell carcinoma D. Hepatoblastoma

The correct answer is A. A. Wilms' tumor is commonly associated with Beckwith-Wiedemann syndrome, a genetic overgrowth syndrome. Other features that may be seen in children with this syndrome include omphalocele, hemihypertrophy, hypoglycemia, large for gestational age, and other dysmorphic features. B. Teratomas are congenital tumors that are present at birth. These benign tumors that are often identified incidentally, or may become symptomatic due to mass effect of the lesion within the abdominal cavity. The aggressiveness of the tumor depends on the degree of differentiation. C. Renal cell carcinomas are much more common in adulthood. Risk factors include cigarette smoking and obesity. D. While children with Beckwith-Wiedemann syndrome can have hepatoblastoma (in addition to other types of tumors), this is not the most common tumor in this genetic condition. Note that hepatoblastoma may also be associated with familial adenomatous polyposis.

A 55-year-old man with no significant past medical history presents for a routine physical exam. He last saw a doctor five years ago. Social history is remarkable for a 35-pack-year tobacco history since the age of 20. He indicates that his wife and children have urged him to quit smoking for the last few months. When you ask him if he has considered quitting, he replies, "I just don't see what the big deal is!" Which stage of change best describes this patient at this time? A. Precontemplation B. Contemplation C. Preparation D. Action E. Maintenance

The correct answer is A. Based on this man's response, it appears he has not actively considered quitting smoking despite his family's concern. All options refer to different stages in the Transtheoretical stages of change model. Given that he has not actively contemplated quitting, the best stage to describe this patient at this time would be the Precontemplation stage and not any of the other responses.

A 15-year-old female with a history of allergic rhinitis presents to the clinic with a five day history of productive cough and wheezing that is worse at night. She denies any shortness of breath, chest pain, or fever. The patient states that she has had similar symptoms in the past, especially when seasons change. The only reported past medical history is atopic dermatitis which is well-controlled with an over-the-counter steroid cream. What is the best next step? A. Prescribe a short acting beta agonist inhaler with a short course of oral steroids B. Prescribe a long acting beta agonist inhaler C. Prescribe a daily corticosteroid inhaler D. Prescribe a daily corticosteroid and long acting beta agonist inhaler E. Prescribe antibiotic therapy

The correct answer is A. Because the patient is symptomatic and wheezing, she needs to be treated with a medication to control her symptoms and treat her exacerbation. A short acting beta-agonist and short course of oral steroids do just that. After controlling her symptoms and treating her exacerbation, a more in-depth interview needs to be performed around the asthma symptoms and other ancillary tests need to be ordered and interpreted including a chest x-ray and pulmonary function test. Then, a decision can be made regarding daily controller inhalers (inhaled corticosteroids, chromolyn, etc). Finally, antibiotic therapy is not indicated in the treatment of an asthma exacerbation.

A 68-year-old male becomes confused and agitated on post-operative day three after open cholecystectomy. He does not recall his surgery or where he is and wants to leave the hospital. Physical exam shows temperature of 99.9 Fahrenheit, blood pressure of 143/89 mmHg, heart rate of 90 beats/minute, respiratory rate of 13 breaths/minute, and O2 sat of 98% on room air. He appears agitated and uncomfortable. He has a Foley catheter. His physical exam is unremarkable. Labs: WBC-11,000, Urinalysis: 2+ nitrites and 2+ leukocyte esterase, 10 WBC/hpf. Glucose finger stick: 80, EKG: normal sinus rhythm at 88 beats per minute. Which of the following is the best initial step in the management of this patient? A. Remove urine catheter B. Give IV ceftriaxone C. Order urine gram stain D. Order urine culture E. Give oral ceftriaxone

The correct answer is A. This patient has an UTI which is the most common cause of delirium in the elderly. Catheters increase the incidence of UTIs and removal would be the appropriate first step. The other options would be appropriate to consider once the catheter is removed.

A 60-year-old woman presents to the office complaining of increased frequency of urination and fatigue for the past several months. She denies fevers, dysuria, back pain, diarrhea and abdominal pain. She has noted some weight loss without working on diet or exercise. Her past medical history is significant for hyperlipidemia and hypertension, for which she takes simvastatin and lisinopril. She is a non-smoker and consumes one to two glasses of wine per week. Her vitals are: Heart rate: 70 beats/minute Blood pressure: 130/70 mmHg Body Mass Index: 30 kg/m2 Physical examination reveals increased pigmentation in her axilla bilaterally. Her labs are as follows: Random plasma blood glucose: 205 mg/dL Creatinine: 0.8 mg/dL TSH: 2.1 U/L. What test is needed to diagnose diabetes mellitus? A. The random blood glucose is sufficient B. Fasting blood glucose C. An oral glucose tolerance test D. HgbA1c E. Urine microalbumin

The correct answer is A. Diabetes can be diagnosed with either an HbA1c > 6.5%, a fasting plasma glucose ≥ 126 mg/dl (7.0 mmol/l), a plasma glucose ≥ 200 mg/dL (11/1 mmol/l) two hours after a 75 g glucose load, or symptoms (such as polyuria, polydipsia, unexplained weight loss) and a random plasma glucose ≥ 200 mg/dL (11.1 mmol/l). Answers (B), (C), (D) and (E) are incorrect, as the diagnosis of diabetes can be made based on random blood glucose with symptoms.

A 27-year-old woman comes into your office because she heard from her friend about a vaccination against cervical cancer and would like one. She has no medical problems and has had a Mirena IUD for three years. She has an allergy to latex and penicillin. She began having sex at the age of 18 and is currently sexually active with one partner. She occasionally uses condoms. She smokes half a pack of cigarettes per day. Her mother had endometrial cancer several years ago and had a total hysterectomy. Why is the patient not an ideal candidate for the Gardasil vaccination? A. Age B. Sexual activity C. Mirena IUD D. Allergy to penicillin E. Tobacco use

The correct answer is A. Gardasil9 is a vaccination against 9 HPV types and is approved for females ages 9 to 26. While the recommendation is to end at age 26, that does not mean it is dangerous; it just hasn't been studied and will not likely be covered by insurance. Activity (B) is not a contraindication to Gardasil vaccination. The other choices are not contraindications to vaccinations.

Ms. Jones is a 35-year-old female with a significant past medical history of SLE who had been on NSAID therapy for the previous three months presented four weeks ago with heartburn. At that time, she reported episodic, mealtime epigastric burning radiating to the throat for the past few months. She has had no surgeries. Serologic testing for H. pylori IgG was reported to be positive a few days after her visit and she was begun on triple therapy. She now returns to the office for follow up. Today she denies any epigastric burning or tenderness. Physical exam is not significant. Which of the following is an accepted indication for performing repeat testing at this visit for H. pylori eradication? Choose the single best answer. A. Restarting of chronic NSAID therapy for SLE B. Documentation is required for all patients with confirmed H. pylori infection C. Treatment with triple rather than quadruple therapy D. Positive serologic test prior to therapy E. Age below 40 years

The correct answer is A. Indications for testing for proof of H. pylori eradication include: patients with an H. pylori-associated ulcer, persistent symptoms despite appropriate therapy for H. pylori, patients with H. pylori-associated MALT lymphoma, history of resection for early gastric cancer, and patients planning to resume chronic NSAID therapy. Documentation of eradication is NOT required for all patients who have tested positive.

A 67-year-old man comes to the clinic for a health maintenance visit. His past medical history is significant for chronic allergic rhinitis, severe chronic obstructive pulmonary disease (COPD), osteoporosis, psoriasis, atrial fibrillation, and benign prostatic hypertrophy. Vital signs show his temperature is 36.8C (98.2F), pulse is 76 beats/minute, respiratory rate is 12 breaths/minute, and blood pressure is 118/70 mmHg. His weight is 129.2 kg (285 lbs) and his body mass index (BMI) is 41. Which of his co-morbidities is most likely to be associated with his BMI? A. Atrial fibrillation B. Benign prostatic hypertrophy C. Chronic allergic rhinitis D. Osteoporosis E. Psoriasis

The correct answer is A. Obese patients are at a significantly increased risk for developing atrial fibrillation. This is thought to be related to increased left-atrial volume. Weight loss may reduce the burden of atrial fibrillation in obese patients. Also, obesity has been associated with COPD and asthma, although the nature of the association has not been fully elucidated. Additional related health risks include atherosclerotic cardiovascular disease (including stroke, coronary artery disease, and peripheral vascular disease) and heart failure. Psoriasis, chronic allergic rhinitis, and benign prostatic hypertrophy are less likely to be associated with obesity. Osteoporosis is associated with low BMI, not obesity.

Ms. Marcos is a 65-year old woman with a past medical history of Type 2 diabetes, hypertension, and hypercholesterolemia who presents with six months of insomnia despite self-medication with acetaminophen, diphenhydramine, and herbal remedies. She is 5' 2" and weighs 250 lbs. When considering a differential diagnosis, which one of the following is a common cause of insomnia in the elderly? A. Sleep Apnea B. Pneumonia C. Chronic sinusitis D. Asymptomatic coronary artery disease E. Hypoparathyroidism

The correct answer is A. Sleep apnea occurs in 20% to 70% of elderly patients. Obstruction of breathing results in frequent arousal that the patient is typically not aware of; however, a bed partner or family member may report loud snoring or cessation of breathing during sleep. Some of the other most common causes of insomnia in the elderly are: Environmental problems such as noise or uncomfortable bedding which are not conducive to sleep. Drugs, Alcohol, and Caffeine such as over-the-counter, alternative, and certain recreational drugs. Parasomnias such as restless leg syndrome/periodic leg movements/REM sleep behavior disorder. In restless leg syndrome, the patient experiences an irresistible urge to move the legs, often accompanied by uncomfortable sensations. In periodic leg movement and REM sleep behavior disorder, the patient experiences involuntary leg movements while falling asleep and during sleep respectively. Disturbances in the sleep-wake cycle such as jet lag or shift work. Psychiatric disorders such as primary depression and anxiety Symptomatic cardiorespiratory disease (asthma, COPD, heart failure) Pain or pruritus Gastroesophageal reflux disease (GERD) due to heartburn, throat pain or breathing problems. Hyperthyroidism The elderly frequently do not present with typical symptoms such tachycardia or weight loss, and therefore further laboratory studies may be required to detect this problem.

Mr. Giovanni is a 37-year-old male who drives a delivery truck. He presents to your clinic after acute onset of severe lower back pain, which began after lifting a large package while at work. When you enter the room, you find him standing, unable to sit comfortably. On physical exam, he has limited lumbar flexion, reduced to 45 degrees, positive straight leg test at 45 degrees on the left, normal gait, but difficulty with heel walk. He has 4/5 strength on the left with ankle plantar flexion. Strength is preserved on the right. Which of additional physical exam finding would be consistent with this man's level of disc herniation? A. Hypoactive ankle tendon reflex B. Decreased range of motion on lumbar extension C. 2/5 strength on hip flexion D. Decreased rectal tone E. Positive Stoop test

The correct answer is A. The clinical signs presented by this patient - difficulty with heel walk and the abnormal strength of ankle plantar flexion - is consistent with nerve root impingement at the level of L5-S1. Of the answers listed, a hypoactive ankle tendon reflex is also consistent with a nerve root impingement at this level. Pain with lumbar extension suggests degenerative disease or spinal stenosis, and spinal stenosis is similarly suggested by a positive stoop test. Diminished hip flexor strength suggests a lesion at the L2, L3, or L4 level and decreased rectal tone suggests a cauda equina lesion.

A 56-year-old male presents for care at the ED complaining of dry cough for the past three days. He notes that this problem started a few days after his family's annual fish fry and barbecue and has been worsening since. He has no known past medical history but mentions that he has not seen a doctor in years. He notes that the cough is worse at night often waking him from sleep. He is unable to lie flat on his back and has started using three to four pillows to sleep comfortably. He also reports increased swelling in his legs that worsens throughout the day. He denies having any chest pain or palpitations and also does not believe he has had any sick contacts. He does not know his family history since he was adopted as a child. He has not had any fevers, sweats, or chills. On exam, you observe a tachypneic, obese man in mild distress. On chest auscultation, he has an S3, bilateral rales at the lung bases, and 2+ pitting edema in the lower legs bilaterally. What is the most likely diagnosis? A. New onset heart failure B. Pneumonia C. Sleep apnea D. Anxiety

The correct answer is A. The patient's description of the cough (worse at night, unable to lie flat, wakes from sleep), physical exam findings (obesity, rales in the lung bases, pitting edema), and history (started after a fish fry/BBQ) point to CHF as the most likely diagnosis. A lack of fever points away from pneumonia and a lack of description of snoring or pauses during sleep points away from sleep apnea. Anxiety would not cause the symptoms or physical exam findings described here.

A 63-year-old male with a past medical history significant for hypertension, COPD, and long-term tobacco use is accompanied by his wife to a hospital follow-up clinic appointment. She is very concerned about her husband's recent hospitalization for a COPD exacerbation and asks what can be done to improve her husband's health. Which of the following holds the greatest long-term health benefit for this patient? A. Cessation of tobacco products B. Immunization against pneumococcus C. Prednisone taken daily D. Pulmonary rehabilitation program E. Tiotropium (Spiriva) inhaled daily

The correct answer is A. This is a key intervention in all patients with COPD who continue to smoke and can reduce the rate of FEV1 decline. The pneumococcal vaccine is recommended for COPD patients ≥ 65 years old or < 65 years old and for all smokers or patients with chronic lung disease. While important to prevent complications from COPD (pneumonia), it is not as fundamental as smoking cessation. Long-term monotherapy with oral corticosteroids is not recommended. Pulmonary rehabilitation may improve dyspnea, walking distance, and quality of life but does not have as much supporting evidence as tobacco cessation. Tiotropium is a medication for COPD, which may be used to decrease symptoms and/or complications, but no medication for COPD has been shown to modify long-term decline in lung function.

A 55-year-old man comes to the clinic for a visit. He has read about the dangers of being overweight and inquires about which category he fits into. He is 5' 10'' (1.78 m) and weighs 220 lbs (100 kg), BMI = 31.6. Which of the following categories most accurately describes the patient based on his BMI? A. Underweight B. Ideal C. Overweight D. Obese E. Morbidly (very severely) obese

The correct answer is D. Based on BMI measurements, Underweight is considered < 18.5; Ideal: 18.5 to 25; Overweight 25 to 30; Obese 30 to 40; Morbidly (very severely) obese > 40.

A 10-month-old asymptomatic infant presents with a RUQ mass. Work-up reveals a normocytic anemia, elevated urinary HVA/VMA, and a large heterogeneous mass with scant calcifications on CT. A bone marrow biopsy is performed. Which of the following histologic findings on bone marrow biopsy is most consistent with your suspected diagnosis? A. Sheets of lymphocytes with interspersed macrophages B. Small round blue cells with dense nuclei forming small rosettes C. Hypersegmented neutrophils D. Stacks of RBCs E. Enlarged cells with intranuclear inclusion bodies

The correct answer is B. A. This is incorrect, as sheets of lymphocytes with interspersed macrophages are associated with Burkitt lymphoma. B. This is the correct response. In addition to neuroblastoma, other tumors associated with small blue cells include Ewing's sarcoma and medulloblastoma, both of which tumors are seen in children. C. Hypersegmented neutrophils are characteristic of megaloblastic anemia, a condition associated with a vitamin B12 and/or folate deficiency, not malignancy. D. Stacks of RBCs suggest rouleaux formation, a phenomenon seen in multiple myeloma, a condition not seen in young infants. E. This describes the classic "owl's eyes" seen in CMV and other viral infections.

A 48-year-old man with a past medical history that includes hypertension, Chronic Obstructive Pulmonary Disease (COPD), and hyperlipidemia presents to clinic as a new patient for a general physical exam. History reveals that he has been smoking a pack of cigarettes daily since age 20. He drinks two beers daily. He is intermittently noncompliant with his medications. Review of the state immunization database reveals that the only immunization he has received as an adult was a tetanus diphtheria shot administered 12 years ago. Which of the following vaccine combinations would be most appropriate for this patient? A. Influenza, Meningococcal, and Zoster B. Influenza, Pneumococcal, and Tdap C. Influenza, Zoster, and Tdap D. Meningococcal, Pneumococcal, and Tdap E. Meningococcal, Pneumococcal, and Zoster

The correct answer is B. Because this man has a diagnosis of COPD and smokes cigarettes, both annual Influenza and Pneumococcal vaccination are indicated. Because his last tetanus immunization was over 10 years ago and because he has not had a booster pertussis shot as an adult, a one-time Tdap is recommended. At this time meningococcal vaccine is recommended for adolescents and young adults and not indicated for this patient. Zoster vaccine is recommended to all adults at age 50 or older.

Ms. Burton is a 45-year-old woman who has never been to a primary care provider. She presents today to establish care and get her health in order. Her concerns today are: fatigue, weakness, numbness, insomnia, feeling sad at times, anhedonia, increased appetite, weight gain, dry skin, and increasing hair loss within the past month. Her vital signs are: Heart rate: 78 beats/minute Respiratory rate: 18 breaths/minute Oxygen saturation: 95% Blood pressure: 152/84 mmHg Weight: 325 lbs Body Mass Index: 41 kg/m2 Today, her physical exam is significant for thinning hair, poor dentition, a systolic murmur heard at the left upper sternal border, an obese abdomen, and bilateral knee stiffness and pain on range of motion exam. Remainder of the physical exam is within normal limits. Which laboratory tests or studies can be done to rule out medical causes of insomnia, fatigue, and depression? A. Chest-X Ray B. CBC, CMP, and TSH C. HgbA1c, lipid panel, urine microalbumin D. CT head without contrast E. MRI brain with contrast

The correct answer is B. CMP can be used to detect electrolyte, renal and hepatic problems. TSH can be used to rule out hypo- or hyperthyroidism. CBC can be helpful to detect anemia and vitamin deficiencies. In addition, ESR can be used to test for rheumatologic disease. An ECG should be done if the patient is using drugs that might alter cardiac conductivity, such as TCAs.

Ms. Anderson is a 60-year-old woman who comes in to clinic as a walk in appointment. She is tearful and is carrying a box of tissues in her hand. She says she doesn't know why but she has been very sad of late. She reports trouble falling asleep and staying asleep. She used to be the head of her Bridge club, but quit two weeks ago and doesn't feel like going out anymore. She also says she has lost interest in walking her dog, and now just allows him to use the doggie door to let himself out. She also says she feels weak and fatigued and no longer has the energy to do her gardening or shopping. She spends most of her day on the sofa crying while watching TV. She also reports a greatly diminished appetite. She denies suicidal or homicidal ideation, but she does have a history of a previous suicide attempt following her divorce seven years ago for which she was hospitalized. A recent CBC, CMP, CXR, TSH, U/A and CT of the head were all within normal limits. How long do the above symptoms need to be present in order to make the diagnosis of Major Depressive Disorder? A. One week B. Two weeks C. Four weeks D. Five weeks E. Eight weeks

The correct answer is B. Depressed mood or anhedonia and at least five of the following eight criteria must have been present for two weeks or longer. (Mneumonic = SIG E CAPS) Sleep: Insomnia or hypersomnia nearly every day Interest (loss of): Anhedonia (loss of interest or enjoyment) in usual activities Guilt: Feelings of worthlessness or excessive or inappropriate guilt (which may be delusional) nearly every day (not merely self-reproach or guilt about being sick) Energy (decreased): Fatigue or loss of energy nearly every day Concentration (decreased): Diminished ability to think or concentrate, or indecisiveness, nearly every day (either by subjective account or as observed by others) Appetite (increased or decreased) Psychomotor agitation or retardation nearly every day (observable by others, not merely subjective feelings of restlessness or being slowed down) Suicidal ideation: Recurrent thoughts of death (not just fear of dying), recurrent suicidal ideation without a specific plan, or a suicide attempt or a specific plan for committing suicide

Ms. H is a 68-year-old woman with a medical history significant for obesity, type II diabetes, hypothyroidism, hypertension, and recently diagnosed hyperlipidemia. Her most-recent lipoprotein (LDL), three months ago, was 197 mg/dL. At that time, atorvastatin was initiated. Other medications include metformin, insulin glargine, amlodipine, hydrochlorthiazide, and levothyroxine. Which of the following may be contributing to her elevated LDL? A. Amlodipine B. Hydrochlorthiazide C. Insulin glargine D. Levothyroxine E. Metformin

The correct answer is B. Dyslipidemia is typically familial, although there are secondary causes of hyperlipidemia that clinicians should be aware of. These include type II diabetes, cholestatic or obstrutive liver disease, nephrotic syndrome, acute hepatitis, alcohol, and medications including hydrochlorthiazide, beta blockers, oral contraceptives, and protease inhibitors. In addition to a possible familial cause of hyperlipidemia, this woman's type II diabetes, hypothyroidsm, and hydrochlorthiazide all may be contributing to dyslipidemia.

A 60-year-old male with a past medical history of chronic gout, depression, and Stage 1 hypertension presents to your office for a follow-up visit. He has been attempting to reduce his blood pressure with behavioral changes, but has had difficulty maintaining the changes. Today, his vitals are blood pressure 144/90 mmHg, pulse 78 beats/min, respirations 12/min, temperature 98.7 F. His recent basic metabolic panel was completely normal. Based on cholesterol levels he had done in the prior week, you calculate his 10-year ASCVD risk at 11%. As you consider starting a medication for his hypertension, which of the following medications is most likely to cause an adverse event in this patient? A. Lisinopril B. Hydrochlorothiazide C. Amlodipine D. Losartan E. Metoprolol

The correct answer is B. Hydrochlorothiazide (HCTZ) can cause hyperuricemia and therefore should be used with caution in patients with gout. Metoprolol is not a first-line choice for the management of blood pressure, but there is no particular reason to expect this patient to experience an adverse drug event due to a beta-blocker. While all of the other medications listed are appropriate first-line anti-hypertensives, many clinicians would select one of the other options over HCTZ for this patient given his history of gout.

Which of the following is not a risk factor for CAD? A. Hypertension B. Female sex C. Smoking D. Diabetes

The correct answer is B. Hypertension, smoking, and diabetes (along with elevated lipid levels, obesity, and sedentary lifestyle) are modifiable risk factors for CAD. Non-modifiable risk factors include older age (men>45 and women >55), family history of CAD in a first-degree relative at a young age, and male sex.

Mr. Turner is a 60-year-old male with a past medical history of hypertension and diabetes who presents with fatigue. Upon further questioning, you realize that Mr. Turner does not have trouble falling asleep but has difficulty staying asleep because he often wakes up short of breath. He also notes that he has swelling in his ankles and feet. He denies having any chest pain or palpitations. On exam, you find that his heart has a regular rate and rhythm. You observe hepatojugular reflux and notice 2+ pitting edema in his legs bilaterally. A recent echocardiogram revealed an ejection fraction of 50 percent. What is the pathophysiology of his condition? A. Death of the myocardial cells due to lack of oxygen leading to dysfunction of the heart B. Decreased compliance of the ventricles leading to dysfunctional filling of the heart C. Decreased forward flow of blood from the heart due to dysfunction of the ventricles D. Dysfunction of the pacemaker cells of the atrium

The correct answer is B. In CHF, decreased compliance of the ventricles leads to increased filling pressures, which leads to congestion. Answer A is describing the pathophysiology of a myocardial infarction, which may lead to CHF, but is not always the cause. Answer C is describing a situation (perhaps an arrhythmia) in which ventricular dysfunction causes abnormal flow of blood from the heart. Answer D is also describing a situation that would lead to an arrhythmia.

A 52-year-old woman comes to the clinic to discuss weight loss. Her medical history is significant for obesity; her body mass index (BMI) is 41; hypertension; hyperlipidemia; and obstructive sleep apnea. She knows that losing weight will help her hypertension and hyperlipidemia, but she doesn't feel like these things bother her. Her only other concern is fatigue; she doesn't use her continuous positive airway pressure (CPAP) machine, because she doesn't like the mask. What additional information can you provide her to help motivate her weight loss? A. Her risk of cardiovascular disease is similar to that of a woman with a normal BMI. B. Her obstructive sleep apnea may improve with weight loss. C. Obesity is mainly a cosmetic issue. D. Surgery should be considered before diet and exercise.

The correct answer is B. Obesity is associated with a number of medical co-morbidities affecting multiple organ systems, including the cardiovascular (atherosclerotic cardiovascular disease, atrial fibrillation, heart failure, venous thromboembolism), pulmonary (obstructive sleep apnea, obesity hypoventilation syndrome), gastrointestinal (gastroesophageal reflux disease, cholelithiasis, hepatic steatosis), endocrine (diabetes), and renal (chronic kidney disease). In patients with medical co-morbidities related to obesity, weight loss is a cornerstone of therapy. In this woman, improvement in hypertension, hyperlipidemia, and sleep apnea can be expected with weight loss. Therefore, counseling regarding lifestyle interventions, weight loss medications, and possibly surgery if non-surgical interventions are unsuccessful, is warranted to manage obesity-related co-morbidities.

A 52-year-old female presents for her third visit this year for productive cough. She has a 34-pack year history but has weaned down to only five cigarettes per day since she began to notice a cough. With her smoking history, you are concerned about the possibility of COPD. Which of the following criteria is included in the GOLD classification for diagnosis of COPD? A. Brain natriuretic peptide >500 B. Post-bronchodilator FEV1/FVC ratio of < 70% of predicted C. Flattened diaphragm on lateral chest film D. Left ventricular function <40% E. Oxygen saturation level of < 89%

The correct answer is B. Spirometry (pulmonary function tests) is the gold standard for diagnosing COPD. If the FEV1 to FVC ratio is less than 70% of predicted (or less than the 5th percentile), then the patient has COPD. Brain natriuretic peptide levels >500 are suggestive of congestive heart failure. Flattened diaphragm on a lateral chest film may be suggestive of advanced COPD but is not diagnostic. Left ventricular function <40% is seen with systolic heart failure. Oxygen saturation levels <89% may be seen in those with COPD but is very nonspecific.

Mr. York is a 44-year-old man presenting for evaluation of an eyelid lesion. He noticed the lesion about one year ago. There is no associated itching, discharge, or other bothersome symptoms. Which of the following is the next-best step in the management of the eyelid lesion? Picture shows eyelid xanthelasma A. Low potency topical corticosteroid B. Measurement of serum cholesterol levels C. Measurement of serum uric acid levels D. No further management E. Skin biopsy

The correct answer is B. The eyelid lesion is most likely a xanthelasma associated with hyperlipidemia. Xanthelasma are cholesterol-filled, soft, yellow plaques which may appear on the medial aspect of the eyelid or on extensor surfaces. They are benign findings, and removal is typically only pursued for cosmetic reasons. Despite the benign nature of the lesion itself, measurement of serum cholesterol levels should be pursued to identify hyperlipidemia in patients with xanthelasma.

A 72-year-old woman with a 30-year history of Type 2 diabetes returns to your office for routine visit. She is taking 20 units of insulin glargine every morning and five units of insulin aspart with meals. The patient notes blurry vision for the past several months and a few days of dark spots in her vision. She denies headaches or nausea. What is true regarding diabetic retinopathy? A. The majority of people with diabetes only develop retinopathy after 10 years with the diagnosis. B. 40% of people with severe diabetes requiring insulin have retinopathy five years after diagnosis. C. Vision changes are an early sign of retinopathy. D. Primary care physicians should examine the retina on every visit for ongoing diabetes care.

The correct answer is B. The patient's symptoms describe diabetic retinopathy. Proliferative retinopathy is prevalent in 25% of the diabetes population with ≥ 25 years of diabetes, but many patients have retinopathy much earlier. Early changes of retinopathy are asymptomatic. Patients need to see an ophthalmologist regularly for a dilated retina exam, not rely on a view of the retina from primary care physicians. Abnormalities seen include macular edema ( a common cause of blurry vision) and new blood vessel formation which can leak and cause dark spots in the vision. Diabetic eye disease often can be treated before vision loss occurs. Glaucoma (causing increased intraocular pressure) is 40% more likely in people with diabetes, but usually causes nausea, headaches, and narrowing of vision or halos around lights.

A 61-year-old male with a history significant for COPD presents to the emergency department for shortness of breath. Upon exam you see a thin male with perspiration on his forehead. He is having a difficult time answering questions because "he just can't catch his breath." You order an arterial blood gas on the patient. The results are pH 7.22 (7.34-7.44) PaCO2 81 mmHg (35-45 mmHg) PaO2 55 mmHg (75-100 mmHg) . What is the next best step in diagnosis or management? A. Administer Rocephin B. Begin noninvasive mechanical ventilation C. Immunize against influenza D. Provide nicotine replacement patches E. Repeat the test in two hours

The correct answer is B. This patient is in respiratory distress as evidenced by his dyspnea, physical exam, and ABG. He has a respiratory acidosis as his pH is low and he is retaining CO2. His PaO2 is also low. Initially you would want to improve his respiratory status with some type of mechanical ventilation such as nasal cannula, facemask, bipap, or even intubation if indicated by worsening of respiratory status such as decreasing oxygen saturation, confusion and drowsiness. After stabilizing the patient, you might consider giving an antibiotic such as Rocephin if he was diagnosed with pneumonia or possibly a COPD exacerbation. Immunizing against the flu and providing tobacco cessation counseling are always good steps in patients with COPD, but would not be the initial step in this scenario. You might want to repeat the ABG after the patient is placed on mechanical ventilation to ensure improvement in his oxygenation status.

A 68-year-old male was diagnosed with Stage 1 essential hypertension a few months ago and has been working on diet and lifestyle modifications. He has a BMI of 28, mild knee arthritis but no other medical diagnoses. He has been a patient of yours for several years, and returns today as planned. Today his blood pressure is 156/94 mmHg. The remainder of his cardiovascular exam is within normal limits. After counseling the patient, he agrees to start an antihypertensive medication. His creatinine is 0.9, urinalysis is normal, and electrolytes are within normal limits. which of the following is the most appropriate medication to begin in this patient? A. Beta blocker B. Thiazide diuretic C. Nitrate D. Loop diuretic E. Clonidine

The correct answer is B. This patient now meets criteria for Stage 2 hypertension as indicated by a systolic BP 140 to 159 mmHg and diastolic BP 90 to 99 mmHg. The ACC/AHA guidelines recommend thiazide diuretics, ACE inhibitors, angiotensin II receptor blockers, or calcium channel blockers as first-line treatment for most patients with newly diagnosed hypertension, with a slight preference for chlorthalidone in the diuretic class. The other options are not first-line treatments for hypertension.

A 65-year-old female presents to your office for a routine visit. She is found to have a blood pressure of 146/96 mmHg. You repeat the blood pressure in her other arm and get 148/92 mmHg. Her pulse is 70 and regular. Her last BP reading was one year ago and was 120/76 mmHg. She has no other medical problems. Her BMI is 28. She states that she likes to walk 30 minutes every other day with her husband and has been doing that for years now. At this time, the most appropriate diagnosis is... A. White coat hypertension B. Elevated blood pressure reading C. Stage 1 hypertension D. Stage 2 hypertension E. Secondary hypertension

The correct answer is B. To diagnose hypertension, two separate readings greater than 130/80 mmHg each time - taken a week or more apart - are needed. Furthermore, ideally home blood pressure readings in the hypertensive range would be needed to confirm that she does not have white coat hypertension. Because this patient has had elevated blood pressure documented on only one occasion (today), the most appropriate current diagnosis is elevated blood pressure. If she has a second similarly elevated reading, Stage 2 hypertension may be diagnosed. Stage 1 hypertension refers to blood pressures between 130-139/80-89 mmHg. This patient has not yet been diagnosed with hypertension, so neither A, C nor D is appropriate.

A 34-year-old woman who has no past medical problems nor is currently taking any medications comes into your office because she noticed a tender lump in her left breast starting approximately one month ago. She is worried because she has an aunt who had breast cancer that was BRCA positive, though her mother is BRCA negative. Her periods have been regular since they started at the age of 13 and occur every 32 days. She is currently menstruating. She has three children aged 12, 9, and 4. On exam, her BMI is 32, up from 28 three years ago and her other vital signs are stable. On breast exam, you note a mobile rubbery mass of approximately 1 x 1cm and with regular borders that is tender to palpation. You appreciate no axillary adenopathy. The rest of her physical exam is unremarkable. Of the information provided, which of the following places this patient at increased risk for breast cancer? A. Age B. Weight C. Parity history D. Family history of cancer E. Age of menarche

The correct answer is B. With a BMI of 32, obesity is the one risk factor for this patient based on the information given. Other risk factors for breast cancer include family history of breast cancer in a first degree relative (mother or sister - not aunt), prolonged estrogen exposure (menarche before age 12, menopause after 45, advanced age at first pregnancy), genetic predisposition (BRCA 1 or 2 mutation), advanced age (breast cancer risk increases with age, and this patient is relatively young), female sex, increased breast density and certain exposures (diethylstilbestrol, hormone or radiation therapy, heavy smoking).

You are seeing a 36-month-old boy for his well-child visit. His parents are anxious about ensuring that his development is appropriate. He passed a hearing screen at birth and, other than a few colds, has been generally healthy. He has never been hospitalized or had any serious illness. He is able to run well, walk up stairs, and walk slowly down stairs. He uses more words than the parents are able to count, but can use them only in short, two or three-word sentences. His speech is understandable. He can draw a circle, but not a cross. Neurologic examination shows normal cranial nerves, normal sensitivity, normal motor reflexes, and no Babinski sign. Which of the following is the most appropriate next step in the management of this patient? A. Perform a brain-stem auditory evoked potential hearing screen B. Perform a screening exam for autism C. Reassure the parents that the boy's development appears normal D. Refer the child to a developmental specialist for comprehensive evaluation E. Refer the child to a specialist for evaluation of his delayed motor development

The correct answer is C. A. A brain-stem auditory evoked potential hearing test (BAER) may be indicated in infants who fail to meet language milestones if they cannot cooperate with other more comprehensive testing. A 36-month-old should be able to cooperate with behavioral audiometry, so a BAER is not indicated. In addition, this child has no evidence of language delay and does not require referral at this point. B. Autism is an increasingly diagnosed cause of developmental delay, but this child is not delayed and no mention is given of any autistic features, such as a lack of symbolic play, repetitive movements, or poor sociability. C. The developmental milestones mentioned in the vignette are within the range of normal for a 36-month-old child. In the absence of any other evidence of significant impairment, there is no indication for referral at this point. D. If there are reasons for concern on developmental screening tests, a referral may be indicated. However, the developmental milestones mentioned in the vignette are within the range of normal for a 36-month-old child. E. This child's motor milestones are not delayed, and no referral is indicated.

Sammy is a healthy male child brought into your office by his mother for a well-child examination. As part of your evaluation you assess his developmental milestones. He is able to run, make a tower of 2 cubes, has 6 words in his vocabulary, and can remove his own garments. What would you estimate Sammy's age to be based upon his developmental milestones? A. 12 months B. 15 months C. 18 months D. 30 months E. 36 months

The correct answer is C. A. At age one year, gross motor skills include pulling to stand, standing alone, and perhaps first steps. Fine motor skills including putting a block in a cup and banging 2 cubes held in hands. At this age a child should be able to imitate vocalizations/sounds and babble. The majority of children this age will know 1 or 2 words in addition to "mama" and "dada." Social-emotional milestones at age one year are waving bye-bye and playing pat-a-cake. Running, building towers of blocks, removing clothing, and a 6-word vocabulary are more advanced skills than a 12-month-old would be expected to have. B. At 15 months of age, a child should be able to stoop and recover and walk well, put a block in a cup, have a vocabulary of a few words, wave bye-bye, and drink from a cup. Running, building towers of blocks, removing clothing, and a 6-word vocabulary are more advanced skills than a 15-month-old would be expected to have. C. At 18 months, a child should be able to walk backward, and 50-90% of children can run at this stage. An 18-month-old should be able to scribble, build a tower of 2 cubes, have 3-6 words in her or his vocabulary, and be able to help in the house and remove garments. D. At 2 ½ years of age, kids can jump up and throw a ball overhand. They can build a tower of 6-8 cubes, point to 6 body parts, name 1 picture, put on clothing, and wash and dry their hands. Sammy is only able to build a tower of 2 cubes, can remove his clothing but does not yet put clothing on, and his vocabulary is limited to 6 words-leading us to believe he is not 2 ½ years old. E. At age 3, children can balance on each foot for 1 second, wiggle their thumbs, name 4 pictures, name 1 color, name a friend, and brush their teeth with help. Sammy's vocabulary is only 6 words, he is not able to name a friend, he is only able to stack 2 cubes, and he has just starting running, but is unable to balance on each foot for 1 second.

A 61-year-old female has recently been diagnosed with Type 2 diabetes. Her fasting glucose was 240 mg/dL and her A1c was 8.9%. Her BP has been 148/90 and 146/86 at two separate office visits. Her home BP measurements have been in a similar range. Her creatinine is 0.9 and she has no known heart disease. She currently takes losartan 100 mg daily for a diagnosis of hypertension. Which of the following would be the most appropriate step in managing this patient's blood pressure? A. Make no changes to her medications as her blood pressure is at goal. B. Start lisinopril daily. C. Start amlodipine daily. D. Start metoprolol daily. E. Start furosemide daily.

The correct answer is C. According to the 2017 AHA/ACC blood pressure guidelines, this patient's blood pressure goal should be 130/80 mmHg. She is clearly above that, and she should have a blood pressure medication added (or in a highly motivated patient, dramatic behavioral changes with close follow up). There is no preference for a first line treatment for blood pressure in diabetic patients, although many providers start with and ACE inhibitor or ARB because diabetes is a risk factor for chronic kidney disease. Furosemide and metoprolol are not among the four major classes of medications for blood pressure management (ACEIs, ARBS, calcium channel blockers, and thiazides), so D and E are not acceptable choices. Lisinopril is an acceptable first-line choice, but it should not be combined with an ARB. Since this patient is taking losartan (an ARB), adding an ACEI is contraindicated. Amlodipine is a good choice for this patient.

A 21-year-old female with no significant past medical history experienced an inversion-type injury to her right ankle while playing soccer a day prior to presentation. She remembers immediate pain and swelling but was able to weight bear and limp off the field. She has noticed some significant swelling which is mostly still present. She has been icing the ankle since the injury as her coach recommended. Pain is still present near the lateral malleolus. Physical exam reveals that the lateral right ankle is edematous with purplish bruising, bilateral pulses are good and the patient had good sensation and motor function in both feet about equally. Palpation of the posterior edge of the lateral malleolus elicits significant pain from the patient. What is the next best step in the management of this patient? A. Reassurance B. Immobilize with cast or splint C. X-ray imaging of right ankle D. Emergent fasciotomy E. Rest, ice, Ibuprofen, compression and elevation and re-evaluate in one week

The correct answer is C. Based on the Ottawa Ankle Rules, tenderness of the lower 6 cm of the posterior lateral malleolus may predict fracture and justifies X-ray imaging of the ankle. Tenderness of the lower anterior lateral malleolus, on the other hand, is very common in ligamentous injury such that x-rays are not warranted. Because of the possibility that a fracture exists, the other treatment options are not correct in this scenario.

Ms. Martinez, 74-years-old, is brought to your family practice by her husband. He is concerned because his wife, who used to take pride in keeping the house in good shape, has stopped cleaning and caring for her appearance. She recently went shopping and left the stove on. She frequently misplaces her car keys and checkbook. Ms. Martinez reports trouble concentrating and does not have much energy. She has a poor appetite and no longer enjoys knitting. Her mini-mental status exam is 28/30. Physical Exam: General Appearance: well-nourished, appears stated age, HEENT[LW1]: pupils equally round and reactive to light, moist mucus membranes, Chest: normal S1 and S2, no murmurs, Lungs: clear to auscultation bilaterally, Abdomen: soft, non-tender, non-distended, Extremities: no cyanosis, no clubbing. Neurologic: no focal deficits Psychiatric: flat affect. Labs: TSH: 2, WBC: 6.0, HgbA1c: 4.5. Which of the following is the most likely diagnosis? A. Normal bereavement B. Dementia C. Depression D. Pseudo dementia E. Delirium

The correct answer is C. Ms. Martinez has a normal mini mental status exam, her exam findings are normal with the exception of her affect. This suggests a mood disorder. ** As there is nothing in the history to suggest a recent loss, bereavement is not likely. History is not suggestive of delirium which would present with more fluctuations in symptoms and alteration in cognition

Ms. Michaels is an 80-year-old female with a past medical history of shingles. She comes to your office accompanied by her daughter Jennifer who reports that her mother is forgetting things. Jennifer explains that her mother will ask the same question several times throughout the day although it was answered. Ms. Michaels also gets confused easily and is more passive than usual. Her memory problem was noticed two years ago after she forgot to pay her bills on multiple occasions. Jennifer now pays her mothers' bills and cleans and cooks for her. Ms. Michaels' vital signs are temperature of 99.2 Fahrenheit, blood pressure of 118/70 mmHg, heart rate of 80 beats/minute, and respiratory rate of 12 breaths/minute. Her physical exam is significant for bilateral osteoarthritis hand deformities. CT head shows mild atrophy of the hippocampus. Her MMSE is 20. The patient's diagnosis is most likely associated with? A. Lewy bodies B. Caudate nucleus atrophy C. Plaque formation D. Prion protein E. Vascular disease

The correct answer is C. Ms. Michaels symptoms and CT results are consistent with Alzheimer's Disease as are plaque formations. Lewy bodies are abnormal aggregates of protein that develop inside nerve cells in Parkinson's disease. Huntington's disease presents with caudate nucleus atrophy. Prion proteins are seen in transmissible spongiform encephalopathies

A 9-year-old boy who moved to the United States from Canada one year ago presents to your clinic with a three day history of fever, sore throat, and muscle aches. He lives at home with his parents who practice strict vegetarianism, and he does not eat meat of any kind. He is up to date on all vaccinations, and review of his medical records indicates that he has had several episodes of otitis media in the last five years. He was hospitalized twice in the past, once for a splenic rupture sustained in a MVA, and the second time was two months ago when he was diagnosed with Kawasaki's disease for which he was put on daily aspirin. You suspect a viral infection and order the appropriate tests. Which of the following is the greatest risk factor for complications of your suspected diagnosis in this patient? A. Recent immigrant B. Splenectomy C. Long-term aspirin therapy D. Age < 10 E. Decreased Vitamin B-12

The correct answer is C. Risk factors for complications of Influenza are children < 5 years, increased in children less than 2 years of age, COPD, conditions that affect the ability to handle respiratory secretions/increased risk of aspiration, CHD, metabolic conditions, Chronic Renal Disease, Immunosuppression, and Long term aspirin therapy. A splenectomy increases his risk of infection by encapsulated bacteria (strep, h. flu etc.), but in this case he probably has influenza so it is not a risk to him now.

A 42-year-old woman presents for a visit after recently being diagnosed with Type 2 diabetes. She has made a plan to work on diet and exercise. Her A1c is found to be 8.0%. What is the best medicine to start at this time? A. A sulfonylurea B. Basal insulin C. Metformin D. GLP-1 receptor agonist E. An SGLT2 inhibitor

The correct answer is C. Sulfanylureas, GLP-1 receptor agonists, and SGLT2 inhibitors may be used as second-line agents. Insulin is generally not used until two other oral medications are insufficient to control the blood sugar, but most people with Type 2 diabetes become insulinopenic over time and require insulin treatment.

A 20-year-old female who is a long-standing patient at this clinic with no significant past medical history presents with first-time onset of dysuria accompanied by frequency and urgency for the past day. She thinks that there is a strange odor to her urine but denies any hematuria. In addition, she feels mild lower abdominal discomfort but denies fevers, chills, nausea, vomiting, constipation, diarrhea, or costo-vertebral angle (CVA) pain. She reports no known allergies She has never been sexually active and has no vaginal discharge nor irritation. LMP was one week ago and was typical. She wonders if using a perfumed bubble bath for the first time a few days ago might have triggered her symptoms. On exam, her vital signs are stable; she has no CVA tenderness and mild suprapubic discomfort. Urinalysis of a mid-stream catch is within normal limits, and a pregnancy test on the same sample, performed despite her history, is negative. Which of the following reflects best management in this situation? A. Insist that a pelvic exam and cervical DNA probe be performed today. B. Await urine culture results before any treatment. C. Reassure that this is not a urinary tract infection. D. Trimethoprim/sulfamethoxazole (one tab twice daily for three days). E. Urine DNA probe

The correct answer is D. There is very good evidence that common typical symptoms of urinary tract infection (UTI) (e.g. dysuria and frequency) with the absence of vaginal symptoms are highly predictive of UTI in young women who have no systemic symptoms. The constellation of typical symptoms outweighs a normal urinalysis. Therefore, based on the symptoms presented, the most reasonable treatment option is to treat empirically with a standard antibiotic for a short course. It is not unreasonable to send urine for culture but management does not need to wait till that result is available. While it is always prudent to consider the possibility of sexually transmitted infection in this age group, based on the information presented about an established patient, it would be inappropriate to insist on a pelvic exam or await results of a urine DNA probe in the face of such strong UTI symptoms.

Mr. Rodriguez is a 32-year-old male who presents with three months of post-prandial epigastric burning. He reports no relief with antacid therapy. He has no allergies and no significant past medical history. He denies any hematemesis, odynophagia, dysphagia, hoarseness, or sore throat. Physical exam is unremarkable other than epigastric tenderness. H. pylori IgG serology is positive. Which of the following treatment regimens will most likely be successful in treating Mr. Rodriquez? Choose the single best answer. A. Omeprazole daily for 8 weeks B. Omeprazole twice daily for 4 weeks C. Omeprazole twice daily, clarithromycin 500mg twice daily, and amoxicillin 1g twice daily D. Omeprazole daily, clarithromycin 500mg twice daily, and tetracycline 500mg three times daily E. Omeprazole twice daily, tetracycline 500mg three times daily, and metronidazole 500mg twice daily

The correct answer is C. The eradication of H. pylori required triple or quadruple therapy. Options include: twice daily dosing of a proton pump inhibitor (PPI) plus amoxicillin plus clarithromycin; a PPI once or twice daily (OR ranitidine twice daily) plus metronidazole, tetracycline and bismuth salicylate four times daily; or, for penicillin-allergic patients, twice daily dosing of a PPI plus clarithromycin plus metronidazole.

Mr. Jones is an 82-year-old man who presents to the office for his six-month chronic disease visit. His diabetes and hypertension are controlled on his usual home medications. He reports that his wife died four weeks ago, and he is now experiencing insomnia most days of the week and fatigue and loss of energy nearly every day; reports decreased enjoyment of his activities, such as playing chess with his neighbor; and is also experiencing loss of appetite but no weight loss. He denies any suicidal ideation and has no previous suicide attempts. Mr. Jones says he often hears his wife's voice while going to bed. He says he goes to church to pray. You are trying to determine if your patient's symptoms are normal grief or if you should diagnose and treat him for Major Depressive Disorder (MDD). Which feature of Mr. Jones' case would suggest MDD rather than a normal grief reaction? A. Insomnia B. Change in appetite C. Inability to experience any joy D. Hearing wife's voice E. Fatigue

The correct answer is C. The loss of a loved one can be a traumatic event and it is normal to experience a period of grief. DSM-5 states that MDD can be diagnosed during a period of grief as long as the criteria are met. Grief can be difficult to distinguish from major depression with symptoms of sadness, fatigue, changes in appetite, sleep disruption, and decreased concentration. Since your patient is also exhibiting diminished pleasure with normally enjoyable activities, this may indicate the patient has MDD, as pervasive unhappiness and misery are rarely a part of the normal grieving process. Other features that differentiate MDD from grief include: Guilt about things other than actions taken or not taken at the time of death Thoughts of death other than feeling that he or she would be better off dead or should have died with the deceased person Morbid preoccupation with worthlessness Marked psychomotor retardation Prolonged and marked functional impairment Hallucinatory experiences other than hearing the voice of, or transiently seeing the image of, the deceased person

A 63-year old woman comes into your office for her annual preventive exam. She has hypertension and type 2 diabetes. She is not sexually active. Her blood pressure is 125/80 and her physical exam otherwise is within normal limits. You recommend influenza and zoster vaccination. Her last colonoscopy was eight years ago and her last mammogram one year ago was normal. She has never had an abnormal Pap smear. At the age of 45 she had a total hysterectomy for fibroids. You tell her she does not require a Pap smear today because: A. She has never had an abnormal Pap smear B. She is not sexually active C. She had a total hysterectomy for fibroids D. She is 63 years old E. She experienced menopause more than 10 years ago

The correct answer is C. The patient described above underwent a total hysterectomy (total removal of the uterus and cervix with or without oophorectomy) for benign reasons (fibroids). USPSTF guidelines recommend against continued cervical cancer screening in patients whose uterus has been removed for benign disease and evidence showed cytologic screening to be very low yield and poor evidence that screening to detect vaginal cancers improves health outcomes in women after hysterectomy for benign disease. Cervical cancer screening should begin at the age of 21 and women between the ages of 65 and 70 who have had three or more normal Pap tests in the past ten years may choose to stop cervical cancer screening. Not being sexually active; age 63; only having had normal PAP smears and years since menopause are not reasons to stop screening for cervical cancer.

A 41-year-old male with no significant past medical history is brought to the Emergency Department after falling to the ground in the middle of a pick-up basketball game with friends. He did not lose consciousness nor hit his head when he fell. As he landed on the ball of his foot after having taken a shot, he recalls hearing a popping sound followed by immediate pain in the posterior right ankle. On physical exam, the posterior right ankle is edematous and palpation is tender. He is unable to plantarflex his right foot. What is the most likely diagnosis of his current condition? A. Ankle ligament sprain B. Calcaneal fracture C. Achilles tendon rupture D. Ankle tendonitis E. Ankle arthritis

The correct answer is C. This is a classical description of an acute rupture of the achilles tendon. Middle-aged males are more commonly affected than other groups. The mechanism does not describe inversion injury making an ankle ligament sprain less likely and there is no direct trauma making fracture less likely. There is no history of overuse or chronicity making arthritis and tendonitis less likely.

A 67-year-old female with an 80 pack year smoking history presents to an urgent care for worsening shortness of breath. She also reports her chronic cough is more frequent and is producing greenish mucous throughout the day. She is using her inhaler every two hours and has been taking an antibiotic left over from a previous sinus infection. On exam there is a whistling noise when she exhales. What other treatment should you consider at this time? A. Digoxin B. Furosemide C. Prednisone D. Propranolol E. Theophylline

The correct answer is C. This patient meets the criteria for a COPD exacerbation as she has increased dyspnea, increased sputum volume and increased sputum purulence. Inhaled bronchodilators (albuterol) and oral glucocorticosteriods such as prednisone are effective treatments with or without an antibiotic. Digoxin is commonly used for patients with congestive heart failure or atrial fibrillation. Furosemide is a diuretic used for a number of things including volume overload. Propranolol is a beta-blocker used for hypertension and coronary artery disease. Theophylline is an oral methylxanthine that antagonizes adenosine receptors and increases cAMP. It can be used in asthma and COPD but would not be the initial choice of drug in this case.

A 55-year-old man with a family history of melanoma presents to the clinic for evaluation of a skin lesion on his back which appeared three months ago. His wife first alerted him to it, hasn't noticed it change and he has not noticed any symptoms associated with it. Physical examination reveals a 7 mm uniformly black macule that is symmetrically round with sharply demarcated borders on his upper back near the right shoulder. Which of the following characteristics would most justify it being biopsied today? A. Symmetry B. Borders C. Color D. Diameter E. Location

The correct answer is D. Using the ABCDE mnemonic, this nevus is not Asymmetrical, does not have irregular Borders, does not display Color variation and he does not describe any Evolution or change or symptoms. The only positive is that its Diameter is > 6 mm, which is considered a red flag supporting biopsy. Location is not considered a predictive factor for melanoma.

A 13-year-old girl comes to your clinic stating she has been having fever and chills for three days, and aching muscles for the last two days. She states she has also had a mild cough, but is not having any difficulty with breathing. She is up to date on vaccines and her only other medical history is having her tonsils and adenoids removed last year. On physical exam, you find her temperature to be 102.6 degrees F, pulse 96, and her BP to be 108/62 mmHg. She has clear rhinorrhea and her oropharynx is mildly erythematous. The rest of her physical exam is normal, and a rapid strep test in the office is negative. What is the next best step in management? A. Zanamavir B. Aspirin C. Ibuprofen D. Amantadine E. Albuterol

The correct answer is C. While this patient's presentation is strongly suspicious for Influenza infection, antivirals such as zanamivir, oseltamivir, amantadine, and rimantadine only decrease the duration of the infection by 24 hours, and are generally not efficacious outside of 48 hours from the beginning of symptoms. This patient also does not appear to require hospitalization at this time, which would be another reason to consider initiating antiviral therapy. Since this patient is presenting outside of the 48-hour window, she should be treated supportively with rest, hydration, and ibuprofen or Tylenol for pain/fever. Albuterol would be unhelpful as there is no history difficulty breathing and no wheezing on exam.

A 40-year-old male presents to the clinic with dry cough and wheezing for the past two days. He states that his symptoms began two days ago with a headache for which he took aspirin. He denies fever, but does report some continued shortness of breath. He is a smoker but only smokes 1-2 cigarettes a day for about six months. Physical exam is negative except for bilateral wheezing and erythema on the face. What is the most likely diagnosis? A. COPD B. Foreign body aspiration C. Pneumonia D. Asthma E. Pulmonary embolism

The correct answer is D. 21% of adults who have asthma have aspirin-induced asthma and should avoid aspirin and NSAIDS. Even though the patient is a smoker, he is less likely to have a COPD exacerbation because he has only smoked for six months. The patient's duration of symptoms (two days) and reporting no fever lead away from the diagnosis of pneumonia. Also, the clinical history gives you very little reason to suspect foreign body aspiration or pulmonary embolism. The diagnosis of asthma could be confirmed with pulmonary function testing.

A 56-year-old male presents for care at the ED complaining of dry cough for the past three days. He notes that this problem started a few days after his family's annual fish fry and barbecue and has been worsening since. He has no known past medical history but mentions that he has not seen a doctor in years. He notes that the cough is worse at night often waking him from sleep. He is unable to lie flat on his back and has started using three to four pillows to sleep comfortably. He also reports increased swelling in his legs that worsens throughout the day. He denies having any chest pain or palpitations and also does not believe he has had any sick contacts. He does not know his family history since he was adopted as a child. He has not had any fevers, sweats, or chills. On exam, you observe a tachypneic, obese man in mild distress. On chest auscultation, he has an S3, bilateral rales at the lung bases, and 2+ pitting edema in the lower legs bilaterally. What diagnostic test would you perform first? A. Exercise stress test B. Pharmacologic stress test C. Echocardiogram D. EKG

The correct answer is D. An EKG can quickly help determine whether the patient is in sinus rhythm, whether there is ischemia or infarction, or whether there is Left Ventricular Hypertrophy. These findings may help you determine the etiology of this patient's CHF. Other tests (especially an echocardiogram) may be indicated at some point during the patient's work-up, but may not be appropriate first tests to order. Specifically, cardiac stress testing is more useful in ascertaining the diagnosis and prognosis of coronary artery disease than in congestive heart failure, although they are often a concurrent part of an extensive workup.

An 18-year-old non-smoking male comes to the clinic with cough, chest tightness, and difficulty breathing. His past medical history is positive only for allergic rhinitis as well as an undiagnosed chronic cough primarily at night since he was 4 years old. He has no past surgical history and no recent travel. On physical exam, you notice the patient appears in mild distress, has hunched shoulders, is using accessory muscles during respiration, and is only able to talk in two to three word sentences. His vitals are: Pulse 125/min, O2 sat 88%, BP 100/70 mmHg. On auscultation, you hear wheezing on inspiration and expiration throughout both lungs. You also notice a prolonged expiratory phase. What is the most likely diagnosis? A. Cystic Fibrosis B. CHF exacerbation C. Foreign body aspiration D. Severe asthma exacerbation E. Pulmonary embolism

The correct answer is D. Based on the clinical presentation including the patient's age and uncomplicated past medical history, the most likely diagnosis is an asthma exacerbation. The presence of hypoxemia places this in the "severe" category. While pulmonary embolism and foreign body aspiration are still on your differential, they are both much further down again based on the patient's age and reported past medical history. Neither cystic fibrosis nor heart failure are likely given the past medical history.

A 65-year-old male with known Type 2 diabetes mellitus presents to the Emergency Department with altered mental status. The patient experienced no known head trauma. His vitals are: Temperature: 38.1 Celsius Heart rate: 102 beats/minute Respiratory rate: 16 breaths/minute Blood pressure: 90/74 mmHg His mucous membranes appear very dry and he is started on IV fluids. Neurological exam reveals no focal deficits. His plasma glucose is found to be 700 mg/dL. Urinalysis reveals no ketone bodies. What is the most likely diagnosis? A. Thiamine deficiency B. Diabetic ketoacidosis (DKA) C. Cerebrovascular accident D. Hyperosmolar hyperglycemic state (HHS) E. Cardiac arrhythmia

The correct answer is D. HHS is seen typically in patients with Type 2 diabetes. It includes very high sugars > 600; ph > 6.4; dehydration; and lack of ketones in the urine and blood. Diabetic ketoacidosis is more common in Type 1 diabetes, and the patient will have ketone bodies in the urine. Thiamine deficiency can cause Korsakoff syndrome, and is typically seen in alcoholics with severe malnutrition, however, this patient is not a known alcoholic and doesn't appear malnourished. Despite the confusion in this patient, stroke is an unlikely diagnosis in this case given the lack of focal deficits on exam. Cardiac arrhythmia can cause dizziness, but is less likely to cause prolonged altered mental status.

Ms. Tsvetanova is a 42-year-old woman with no significant past medical history presenting to establish care with her primary care physician. On review, she notes a weight gain of 14 kg (30 lbs) over the last three years. She attributes this mostly to her sedentary lifestyle, snacking, and difficulty with portion control. She works as a receptionist for a local physician's office and spends most of her day sitting. She denies constipation, low energy, cold intolerance, muscle weakness, depressed mood, easy bruisability, or other skin changes. On physical exam, vital signs reveal temperature is 36.8C (98.2F), pulse is 82 beats/minute, respiratory rate is 12 breaths/minute, blood pressure is 130/82 mmHg, weight is 81.6 kg (180 lbs), and height is 163 cm (64 in). The remainder of her physical exam is normal. Which of the following laboratory tests is most appropriate for the evaluation of this patient? A. 24-hour urine catecholamine levels B. 24-hour urine cortisol level C. Basic metabolic profile D. Lipid profile E. Thyroid stimulating hormone (TSH)

The correct answer is D. In the absence of symptoms and signs associated with secondary causes of obesity such as hypothyroidism and Cushing's disease, the initial laboratory evaluation in patients with obesity should be limited to assessment of co-existing risk factors for atherosclerotic cardiovascular disease (ASCVD), including dyslipidemia and diabetes. Screening guidelines vary regarding lipid screening in adults. The American Heart Association (AHA) recommends screening all adults aged 20 to 79 years old every four to six years in those without ASCVD. The U.S. Preventive Services Task Force (USPSTF) strongly recommends (grade A recommendation) screening in all men >35 years old and women >45 years old who are at increased risk for heart disease. The USPSTF also recommends (grade B recommendation) screening for men 20 to 35 years old and women 20 to 45 years old if they are at increased risk for heart disease.

A 47-year-old woman comes into your office for a health care maintenance exam. She has hypertension and type 2 diabetes. She is not sexually active and has not yet experienced menopause. There is no family history of cancer. Her blood pressure is 118/78, her BMI is 34 and the remainder of her physical exam is within normal limits. Her vaccinations are up-to-date, she has a PAP smear today and will have labs drawn. According to USPSTF, which of the following is the best recommendation to give her concerning mammography? A. Should have started at age 40 and every year thereafter B. Should have started at age 40 and every 2 years thereafter C. Start at age 50 and every year thereafter D. Start at age 50 and every 2 years thereafter E. Should have started at age 45 and every year thereafter

The correct answer is D. Mammography has a sensitivity of 60% to 90% for detecting breast cancer and decreases breast cancer mortality. According to the most recent USPSTF guidelines, routine mammography is not indicated for women younger than 50 years old except as based on patient context (history) and beliefs about risks/benefits. The USPSTF recommends biennial testing for women between the ages of 50 and 74. There is insufficient evidence to assess the benefits versus risk of screenings in women after the age of 75. Other groups such as the American Cancer Society (ACS) and American College of Obstetricians and Gynecologists (ACOG) recommend yearly mammograms starting at age 40, continuing as long as the woman is in good health.

Ms Smith is a 38-year-old female with a significant past medical history of hypercholesterolemia who presents to her primary care physician after 4 weeks of episodic epigastric discomfort. She reports a recurrent ache-like sensation in the epigastric region occurring about 10 minutes after eating and lasting for several hours. The pain does not appear to be associated with any particular food. She denies any odynophagia or dysphagia. Ms Smith is a smoker, with a 30-pack year history. You are concerned about peptic ulcer disease. Which additional history finding would increase your level of concern? Choose the single best answer. A. Report of hoarseness B. Epigastic discomfort associated with recumbency or bending C. Bouts of recurrent laryngitis D. Return symptoms despite consistent use of antacids E. Sour or bitter taste in mouth

The correct answer is D. Symptoms that would increase the likelihood of PUD include pain that improves with meals (although some people experience the opposite), history of NSAID use, a pain described as "gnawing" or "hunger-like" and persistent symptoms despite adequate acid-blocking therapy. Hoarseness, epigastric discomfort with lying down, laryngitis, discomfort after large meals, and bitter taste are all typical symptoms of GERD.

It is November and you are working in a small, rural, emergency department serving a community who is currently going through a flu epidemic. Your next patient is a 4-year-old boy who was brought in by his mother for a sore throat and fever that started two nights ago. She says he has a mild cough, and is complaining of headaches as well. Since last night, he has had a decreased appetite and hasn't been his normal, active self. She also wants you to know that he is allergic to eggs and latex, and uses an inhaler once a month for asthma like symptoms. On physical exam you note an erythematous throat, clear rhinorrhea, and rhonchi on auscultation. A rapid strep test was performed in the office and is negative. His last well child check was 14 months ago, and his mother says she knows he is due for another but her schedule has been too busy. What is the next best step in management? A. Flu Shot B. Amoxicillin therapy C. FluMist (influenza vaccine, live intranasal) D. Tamiflu (oseltamivir phosphate) E. Influenza test.

The correct answer is D. Tamiflu can be prescribed in patients who show signs of flu like symptoms without further testing in communities with known flu outbreaks. Tamiflu can reduce symptoms of the flu, and is recommended only in patients who are still within a 48 hour window since symptoms began. In a population that has a known flu outbreak, patients presenting with symptoms compatible with the flu do not require a nasal swab for influenza unless it will change the treatment. Amoxicillin could be a potential answer if you were treating strep throat, but a negative strep test and only moderate probability make this a less likely choice. The Flu Shot is not recommended in patients with active illness/fever. While research has shown that the flu shot can be given to patients with egg allergies, the live virus form of the vaccine given in Flu mist should be avoided in this population. Also, Flu Mist is currently not approved for patients with respiratory disease such as asthma.

Ms. Vasquez is a 38-year-old female with a past medical history of sarcoidosis and recently completed a six-month steroid taper. She presents to her primary care physician after two weeks of lower lumbar back pain. She does not recall any trauma but began to feel a sharp pain after bending over to pick up laundry. The pain radiates bilaterally into her anterior abdomen. She has found no relief with over the counter NSAIDs. On physical exam, she has point tenderness along her vertebrae in the L1-L2 region. There are no neurologic deficits and reflexes are intact. Which of the following is the most appropriate next step in management? A. Reassess in four weeks B. Refer to spine specialist C. Order complete blood count D. Order a plain x-ray E. Recommend conservative management

The correct answer is D. The "red flag" in this history is that of chronic steroid use, and the concomitant risk of osteoporotic vertebral fracture. A vertebral fracture is best diagnosed with a plain x-ray. A CBC will not help with the diagnosis, and referral to a spine specialist is unnecessary. Conservative management and/or reassessment in four weeks demonstrate a failure to recognize the "red flag."

Ms. Rogers is a 75-year-old woman who was found unresponsive in her house by her neighbor who had come over to help clean her house. An empty unlabeled pill container was found next to her on the bathroom floor. She was rushed to the ER, stabilized and is now in ICU on a mechanical ventilator. Which of the following are true regarding suicide in the elderly? A. Elderly persons attempting suicide are more likely to be married and living with their spouse. B. Elderly persons attempting suicide usually report good sleeping habits. C. Suicidal behaviors increase with age, but rates of completed suicides don't. D. Approximately 75% of the elderly who commit suicide had visited a primary care physician within the preceding month, but their symptoms went unrecognized. E. Firearms are the most common means of suicide in the elderly.

The correct answer is D. The USPSTF recommends screening all adults for depression, but especially patients with chronic diseases like diabetes, as they are at high risk for depression. The PHQ-2 inquires about the frequency of depressed mood and anhedonia over the past 2 weeks. The purpose of the PHQ -2 is not to establish a final diagnosis, but rather to screen for depression as a "first-step" approach. Patients who screen positive should be evaluated by the PHQ-9 to determine whether they meet the criteria for depression. Another screening tool which can be used is the Geriatric Depression Scale - Short Form (GDS-SF) which includes a series of 15 questions. Specifically related to suicide in the geriatric population: Elderly persons attempting suicide are more likely to be WIDOW(ER)S, AND LIVE ALONE; Elderly persons attempting suicide have REDUCED sleep quality; Suicidal behaviors DO NOT increase with age, but rates of completed suicides DO. Drug overdose is the most common means of suicide in the elderly.

A 62-year-old woman presents for follow-up of her hypertension and diabetes. In general, her chronic diseases are well controlled and she has suffered no target organ damage. She has worked hard to begin exercising, and is walking vigorously five times a week. She has also worked hard on dietary changes, and has been following the DASH eating plan very seriously. She quit smoking three months ago. Her blood pressure today is 148/88 mmHg, pulse is 72 and BMI is 32. She is taking metformin 500 mg twice daily, simvastatin 20 mg daily and hydrochlorothiazide (HCTZ) 25 mg daily, and she is compliant with her daily medications. Her labs today include an A1C of 6.6, an LDL of 88 and a basic metabolic panel within normal limits. Which of the following management steps today do you consider the most appropriate? A. Increase HCTZ to 50 mg daily B. Make no changes as she is at her treatment goals C. Impress upon her the importance of making more lifestyle modifications D. Add amlodipine 5 mg daily E. Change her simvastatin to atorvastatin 20 mg

The correct answer is D. The goal blood pressure for patients with hypertension is 130/80 mmHg, and this patient has not met this goal with HCTZ and major lifestyle changes. Increasing the dose of HCTZ from 25 to 50 does not improve blood pressure further, so adding a second medication would be more beneficial. While commending her on her lifestyle changes is important, counseling about intensifying them is not likely to be realistic nor helpful given all that she has already done. There is no need to change her statin, however calculating her ASCVD risk to determine whether she is on the appropriate dose would be helpful. The current cholesterol guidelines recommend a moderate intensity statin for patients with diabetes. For simvastatin, a dose of 10 mg represents a low-intensity dose. Increasing this to 20 mg would put her management more in line with these guidelines, though it would not address her elevated blood pressure.

A 22-year-old female with no significant past medical history experienced an inversion-type injury to her right ankle while playing volleyball. The ankle quickly became edematous, but she used ice and was able bear weight on the foot. When she comes to clinic two days later, there is mininmal edema, she has good motor function, and has normal sensation. She has tenderness at the anterior lower lateral malleolus but not inferiorly nor posteriorly. X-rays are not indicated. You recommend continued relative rest and also tell the patient to keep it elevated and ice it several times during the day to help with the pain and swelling. You inform the patient that immobilization and compression is good for the conservative management of her condition. What is the best compression device to use in this situation? A. Tape B. Compression stockings C. Elastic wrap D. Semi-rigid ankle support E. Solid cast

The correct answer is D. This patient apparently has a ligamentous injury to her anterior talo-fibular ligament. In recommending the RICE mnemonic, a semi-rigid ankle support (like an Air Stirrup) provides protection from repeat inversion injury while allowing the patient to actively dorsi- and plantar-flex her foot, which aids recovery. Compression stockings and elastic wrap do not provide adequate support. A solid cast completely immobilizes the ankle and delays recovery, while the evidence supporting taping of the ankle is lacking.

Sally is a generally healthy 27-year-old female graduate student who presented to your office twelve weeks ago with episodic post-prandial epigastric burning. This had been bothering her for nearly six months but she had been busy with her thesis and was unable to find the time necessary for an appointment. She reported this year has been particularly stressful, with limited time resulting in increased consumption of coffee and take-out fast food. At that time, she began an eight-week trial of omeprazole. She returns now with no improvement of her symptoms. She discontinued the omeprazole one month ago because she ran out of the medication. She has no additional symptoms and physical exam is unremarkable. Which of the following is the most appropriate next step in her management? Choose the single best answer. A. Refer her for an upper gastrointestinal endoscopy B. Begin treatment with triple therapy of pantoprazole, clarithromycin, and azithromycin for 14 days C. Continue omeprazole for another 4 weeks with close follow up D. Administer a urea breath test E. Switch PPI from omeprazole to pantoprazole

The correct answer is D. This patient exhibits no alarm symptoms, but does have persistent symptoms despite adequate empiric therapy. This case warrants testing for H. pylori as a cause of symptoms. One test which is sensitive and specific for H. pylori infection is the urea breath test. The antibiotic regimen listed in choice B is for treatment of H. pylori, but a diagnosis must be made before instituting such a treatment regimen. Switching PPI's or doing a longer treatment of the same medication would not be optimal choices, since the patient hasn't found any relief thus far with the medication. While an upper endoscopy might assist with diagnosis, it is more invasive and in the absence of alarm symptoms, a less invasive test for H. pylori (urea breath test) is acceptable.

A 28-year-old male comes to the Emergency Department with shortness of breath, cough and wheezing for the last 4 hours. He states that he was diagnosed with asthma recently, and is currently using inhaled corticosteroid with a long acting B2-agonist daily to control his symptoms. His RR is 34/min; Temp: 98.8 degrees F; O2sat: 88%; BP: 130/85 mmHg. What treatment should be given to this patient first? A. Albuterol breathing treatment using a nebulizer B. Intravenous corticosteroids C. Ipratropium breathing treatment using a nebulizer D. Oxygen E. Intravenous third generation cephalosporin

The correct answer is D. This patient is in acute respiratory distress with a decreased oxygen saturation and increased respiratory rate. The first treatment this patient should receive is oxygen supplementation. After giving the patient oxygen, you can then continue treatment for his acute asthma exacerbation with albuterol and ipratropium breathing treatments and intravenous or oral corticosteroids. Antibiotics are not indicated for treatment of asthma exacerbations.

A 54-year-old male with a history of chronic gout and GERD presents to your office for his health maintenance exam. Vitals today are blood pressure 138/88 mmHg, pulse 65 beats/min, respirations 10/min, afebrile, BMI 29 kg/m2. He smokes 10 cigarettes per day, does not regularly exercise, and drinks one to two beers daily, four or five times a week. He has no current concerns, review of systems is negative, and his physical exam is unremarkable. You recommend lifestyle changes. Which of the following changes is least likely to improve his blood pressure? A. Smoking cessation B. DASH eating plan C. Weight loss D. Alcohol cessation E. Increased exercise

The correct answer is D. While all of these options are reasonable lifestyle modifications to recommend for patients, this particular patient's blood pressure is least likely to be reduced by alcohol cessation. Moderate alcohol consumption actually improves blood pressure by 2 to 4 mmHg; therefore, stopping his moderate alcohol consumption could increase his blood pressure. However, it is not recommended to encourage alcohol use in patients who do not drink, because of the risk of encouraging problem drinking. Initiation of the NIH-sponsored Dietary Approaches to Stop Hypertension (DASH) eating plan has been shown to lower systolic pressure as have smoking cessation, weight loss, and increased exercise.

Mr. Jones is a 63-year-old male with no significant past medical history who presents with increased swelling in his legs and shortness of breath. He reports that he has smoked two packs per day for the past 20 years. He does not exercise regularly and notes that he has always been overweight. Mr. Jones notes that his maternal uncle died of a heart attack at age 43. He denies having any chest pain or palpitations and his exam is notable only for 2+ pitting edema in the lower extremities. His most recent labs show HDL of 50, LDL 101, and a blood glucose of 112. Which of the following risk factors has most likely contributed to Mr. Jones' problem? A. Diabetes B. Elevated HDL C. Heart attack in uncle D. Male sex

The correct answer is D. With a blood glucose of 112, he is in the pre-diabetic range, but does not have a formal diagnosis of diabetes. His HDL is on the high side, but this is a protective factor. His uncle, though he did have a heart attack at a young age, is not a first-degree relative.

An asymptomatic, healthy 9-month-old female is found to have a palpable RUQ mass on exam. After further imaging and lab studies, the mass is diagnosed as a neuroblastoma that has involvement in the bone marrow as well. The mother is worried about the prognosis. Which of the following is true about the prognosis of neuroblastoma in this child? A. Lymph node involvement is a poor prognostic factor B. Prognosis of neuroblastoma is predictable C. Children who are older than 12 months have a better prognosis than younger children D. Favorable histology does not play a role in prognosis E. Non-amplification of the n-myc gene is a favorable prognostic factor.

The correct answer is E. A. Due to the effectiveness of chemotherapy, neuroblastomas with lymph node involvement are still considered favorable, especially in the setting of other favorable factors, such as young age and differentiating histology. Though distant metastasis is a significant poor prognostic factor, regional lymph nodes do not significantly affect the outcome. B. Neuroblastoma has a broad spectrum of clinical courses. Some tumors may spontaneously regress, some may mature to a benign type, and yet other tumors can be very aggressive with metastases. Age plays a role in the prognosis, as most infants have a good prognosis even with disseminated disease, while infants over 18 months of age do not do as well. C. In infants less than one year of age, neuroblastoma tumors may spontaneously regress. Stage 4S neuroblastoma is a special category that is reserved for infants less than 12 months who have resectable primary tumors and metastases to the liver, skin, and bone marrow. Overall survival is over 85 percent for babies over 6 weeks of age with Stage 4S. D. Favorable histology is a good prognostic factor in neuroblastoma, and is based on the differentiation of the cells involved. E. Non-amplification of the n-myc gene is one of the favorable genetics in neuroblastoma.

A 68-year-old male with GOLD Stage 3, Group D, COPD requiring 2L of oxygen at nighttime presents to clinic complaining of increasing lower extremity edema over the past few weeks. He also thinks his nighttime cough might be worse. His physical exam reveals distant breath sound with scattered rhonchi, a normal cardiac exam, and 2+ bilateral pitting edema in his legs up to his mid shins. What is the most likely mechanism of disease underlying his lower extremity edema? A. Decreased blood flow to the lower extremities due to thromboembolism B. Hepatomegaly from infiltration of the liver with granulomas C. Irregular heart rate due to atrial fibrillation D. Overexpansion of lower extremity veins due to incompetent venous valves E. Pulmonary hypertension causing right heart failure

The correct answer is E. Chronic hypoxia causes pulmonary vasoconstriction that increases blood pressure in the pulmonary vessels. This elevation in blood pressure causes permanent damage to the vessel walls and leads to irreversible hypertension. The right heart eventually fails because the pump cannot sustain flow effectively against this pressure. Right heart failure leads to an increase in preload, with peripheral edema and increased jugular venous distention. Given this patient's history of advanced COPD, E is the most likely cause. Thromboembolism would cause painful necrotic areas in the distal extremities but is not a common cause of peripheral edema. Hepatomegaly can cause venous congestion, but there is nothing in this patient's history to suggest this is likely. New onset atrial fibrillation can cause left-sided heart failure and dependent edema. This patient's cardiac exam is described as normal which indicates he is not in atrial fibrillation currently (though he could have paroxysmal AFib). He also does not present with lung findings suggestive of pulmonary edema such as crackles. His lung findings are typical for a patient with COPD. Venous stasis from incompetent venous valves is a common cause of dependent edema. This is frequently the result of a deep vein thrombosis, and thus is more typically unilateral. In this patient with advanced COPD, pulmonary hypertension is a much more likely explanation.

Working at your clinic, you receive a call from a patient of yours, Mr Smith, a 45-year-old male who was seen three days ago complaining of lower back pain. At that time he had no history of trauma, pain that improved while lying down and no neurologic deficits. He works as a truck driver. He was treated conservatively along with pharmacologic intervention with NSAIDs and muscle relaxants. He calls your office now due to only minimal improvement. And although his symptoms have not changed, he is frustrated with the slow progress, needs to get back to work as soon as possible, and is concerned this might be "something serious." Which of the following is the most appropriate next step in management? A. Obtain a plain film x-ray B. Order an MRI C. Ask him to double the dosage of his muscle relaxants D. Schedule him for an appointment immediately E. Reassure him and schedule a follow-up appointment in a few days

The correct answer is E. Given this clinical presentation, the likelihood of this being an episode of lumbar sprain/strain is high, and the odds of this being "something serious" (nerve root compression, malignancy, infection) is still low. The original plan is a good one, and should continue. No new meds or imaging studies would help, and an urgent appointment will not change the anticipated course. Some physicians might choose to involve a physical therapist at this time, but this option is not available for this question.

A 7-year-old boy is brought to your clinic with a fever of 102-103° F (38.9-39.4° C) for the last three days. He is up-to-date on all vaccinations and has no significant medical history. His mother notes that he has not had a cough but is eating and drinking less because "it hurts to swallow." On examination of his neck you notice tender cervical lymphadenopathy bilaterally, and auscultation of his back shows clear lung sounds on both sides. His oropharyngeal exam shows erythematous throat, but no tonsillar exudates. What would be the most appropriate next step? A. Empiric Amoxicillin therapy B. Empiric Levofloxacin therapy C. Empiric Tamiflu therapy D. Chest x-Ray E. Rapid Strep Test

The correct answer is E. In addressing this child presenting with fever, symptoms of pharyngitis (sore throat), and the absence of cough, the physician needs to consider the possibility of strep throat. Predictive scoring tools such as the Modified Centor criteria are useful in determining which patients need testing for strep, but should not be used to make a positive diagnosis of strep throat. This child would have a score of 4 (one each for fever, absence of cough, high-risk age group, and tender cervical lymphadenopathy) and should be tested for strep using a rapid strep test. Empiric antibiotics are not appropriate here. In the case of a positive strep test, amoxicillin would be an appropriate choice, but levofloxacin would not. Influenza often presents with fever and sore throat, but typically involves cough as a prominent symptom. This patient has no lung findings, so a chest x-ray is unlikely to be of benefit.

A 48-year-old female with no smoking history comes to the emergency department with complaints of sore throat for the last two days. She does not have a thermometer at home, but states she has been feeling hot and her children have also been out from school for a fever and sore throat. Her children are now staying with their father who she is separated from, and he notified her yesterday that they were on antibiotics for their symptoms. She was holding off coming to the doctor because she wasn't coughing or having any problems with swallowing until breakfast this morning. You note that she has a fever of 38.6 degrees Celsius measured by the nurse, and on physical exam you observe an erythematous throat with exudate, and bilateral cervical lymphadenopathy. What is the next best step? A. Amoxicillin therapy B. Levofloxacin (Levaquin) therapy C. Tamiflu therapy D. CXR E. Rapid Strep Test

The correct answer is E. Modified Centor Criteria (also called McIsaac Score) allows you to calculate the need for various interventions in a patient who presents with signs/symptoms of strep throat. One point is assigned each for fever greater than 38.0 degrees Celsius, absence of cough, tonsillar exudates, cervical lymphadenopathy, and age less than 15 years old. One point is removed for age older than 45 years old. Her total points is 4 for temp, cervical lymphadenopathy, no cough, and tonsillar exudates, minus 1 point for age < 45. This gives you a total score of 3, indicating the need for rapid strep test to guide further treatment.

Mr. Brown is a 42-year-old male accountant with a significant past medical history of obesity who presents to his primary care physician after one week of lower back pain. After moving into a new home three days ago, he woke up the next morning with bilateral lower back pain without any radiation. He denies any recent trauma, fever, chills, numbness, tingling, or incontinence. He has not had any urinary frequency or dysuria. He takes no medications and has no significant past medical history. Which additional findings in his history or physical exam would make the diagnosis of lumbosacral sprain/strain more likely? A. Increased pain with coughing B. Abnormal gait C. Point tenderness on spinous processes D. Loss of ankle jerk E. Spasm of paraspinous muscles

The correct answer is E. Spasm of the paraspinous muscles suggests lumbosacral sprain/strain. Increased pain with coughing, abnormal gait and loss of ankle jerk point to conditions that compress a regional nerve root, while point tenderness on the spinous processes often indicates an origin in the vertebra (osteoporotic fracture, malignancy, etc.).

Dr. Rodriquez, your family medicine preceptor, is giving a talk about ways to prevent delirium in patients during their hospital stay. What did she most likely suggest? A. Only allow the patients visitors to come at night B. Give diazepam every six hours C. Keep the patient in restraints D. Keep the patients room quiet E. Keep the patients room well lit

The correct answer is E. Stimulation such as a well lit room with activity has been shown to decrease delirium in hospitals. Limiting visitors and keeping the room quiet could increase the risk of delirium. Benzodiazepines such as diazepam can bring on delirium. Restraints would be a response

A 55-year-old man with no significant past medical history and generally healthy behaviors presents to clinic for a health care maintenance exam. He says, "I'd like to get tested for all types of cancer." He does not have any family history of cancer. Review of systems is negative for any symptoms of prostate cancer, such as urinary frequency, urgency, retention, hematuria, weight loss, or back pain. He is a lifelong non-smoker, and he doesn't drink alcohol or use recreational drugs. Which of the following screening tests is given either an A or B recommendation in favor of its routine use for patients such as this one? A. Prostate Specific Antigen (PSA) testing B. Lung cancer screening C. Pancreatic cancer screening D. ECG screening for coronary artery disease E. Colon cancer screening

The correct answer is E. The USPSTF gives colon cancer screening an A recommendation for people age 50 to 75 years due to clear evidence of benefit. Lung cancer screening is given a B recommendation for 55-year-old men with a 30 pack-year tobacco history and who have smoked in the past 15 years. This patient is a non-smoker. Pancreatic cancer screening and ECG screening are both given D recommendations (against their use). PSA screening is given a C recommendation, indicating that doctors and patients should make individualized decisions about the use of this test.

A 64-year-old woman who is overweight with well-controlled hypertension comes to your office complaining of a lump in her breast that she noticed while showering. She denies any pain, tenderness, or skin changes. A pertinent review of systems is negative. Menarche began at the age of 10. Her first child was born when she was 31 and she had her second and last child at the age of 33. She experienced menopause at the age of 44. Her mother died of colon cancer when she was 65 and her father passed away from metastatic prostate cancer at the age of 70. She has no history of tobacco use ever and occasionally drinks a glass of wine with dinner. Her BMI is 34. Which of the information provided thus far puts the patient at decreased risk for breast cancer? A. Age B. Weight C. Age at first birth D. Age at menarche E. Age at menopause

The correct answer is E. The patient experienced menopause at the age of 44, which shortens her time of estrogen exposure, a known risk factor for the development of breast cancer. Factors associated with decreased breast cancer risk include pregnancy at an early age, late menarche, early menopause, high parity and medications such as selective estrogen receptor modulators along with NSAIDs and aspirin. Risk factors for breast cancer include family history of breast cancer in a first degree relative (mother or sister), prolonged estrogen exposure (menarche before age 12, menopause after 55, advanced age at pregnancy, obesity after menopause), female sex, genetic predisposition (BRCA 1 or 2 mutation), advanced age (breast cancer risk increases with age), increased breast density and exposures (diethylstilbestrol, hormone or radiation therapy, smoking).

A 19-year-old female with no significant past medical history is involved as the driver in a motor vehicle accident and brought to the Emergency Department by EMS. She is complaining of severe pain in her right lower extremity that has been worsening since the accident. In addition, she has started to notice what she describes as "burning and tingling" in her right foot. On physical exam, her right calf is edematous and tender with tense overlying skin. There is no swelling or tenderness of the right foot or ankle but the right dorsalis pedis and posterior tibial artery pulses are barely palpable. She cannot confirm light touch of the foot and cannot wiggle her toes on command. What is the next best step in the management of this patient? A. Reassurance and icepacks q 2 hours B. Immobilize leg and ankle with a cast C. Urgent EMG of the right lower extremity D. Diagnostic imaging of right foot and ankle E. Emergent fasciotomy

The correct answer is E. This clinical scenario describes acute compartment syndrome which is a vascular emergency. Emergent fasciotomy is the treatment of choice to relieve pressure in the calf and, if not performed, the limb could be lost due to acute ischemia. While emergent radiographs of the tibia and fibula are appropriate to evaluate for co-existent fracture, x-rays of the foot and ankle are not indicated. Reassurance, ice packs, urgent EMG and immobilization are all incorrect treatments and place the patient at risk of serious permanent adverse outcome.

Mr Gill is a 27-year-old male who presents to his primary care physician with post-prandial epigastric burning. The burning is episodic, without associated hematemesis, dysphagia, or odynophagia. In the past, he has episodically used an OTC chewable calcium carbonate to provide relief. Over the past month, he has had to increase the frequency of its use to four times daily. His blood pressure is 120/71 mm Hg, heart rate at 75/min and regular. Physical exam reveals only minimal epigastric tenderness on palpation but is otherwise normal. Which of the following is the most appropriate next step in management? Choose the single best answer. A. Upper endiscopy B. H. pylori IgG serologic testing C. Fecal immunochemical testing (FIT) D. Fecal occult blood testing (FOBT) E. 8 week trial of pantoprazole

The correct answer is E. This patient displays no red flag symptoms at this time. The most widely accepted initial intervention in a patient like this is empiric treatment with a histamine-2 receptor blocker or a proton pump inhibitor such as pantoprazole. Upper endoscopy would be indicated if there were alarm symptoms or if empiric therapy did not resolve symptoms. H. pylori testing might be indicated if symptoms persisted after empiric therapy. There is not a specific concern for rectal bleeding at this time, so fecal testing for blood is not indicated.

Ms. Brady, a 78-year-old female prevents to your office after six months for follow-up. Her interval history is significant for a TIA three months ago. Today her MMSE is 19. You note that six months ago her MMSE was 22 and nine months ago it was 26. Physical exam shows temperature of 98.8 Fahrenheit, blood pressure of 167/95 mmHg, heart rate of 76 beats/minute, and respiratory rate of 14 breaths/minute. Chest: regular rate and rhythm, no murmurs. Lungs: clear to auscultation. Neuro: weakness in the right upper extremity. Abdomen: soft, non-tender. She takes atorvastatin and aspirin. Lab studies show Hgb A1c: 5, TSH: 3, B12: 500 pg/mL. Which of the following is the most important recommendation to prevent further disability in this patient? A. Nothing can be done B. Start metformin C. Start vegetarian diet D. Weight loss E. Start hydrochlorothiazide

The correct answer is E. To prevent additional TIAs and stroke, her blood pressure needs to be controlled. Hydrochlorothiazide is a reasonable first line medication. Her HgbA1C is in the normal range so metformin would not be an appropriate choice. A vegetarian diet would have little impact on limiting dementia, and we don't know what she weighs and whether weight loss would be appropriate.

Mr. Roberts is a 78-year-old male with a significant past medical history of chronic kidney disease stage II, coronary artery disease, and hypertension who presents lumbar back pain. He has also been feeling general malaise and chills over the past few days. On review of symptoms he reports having some difficulty urinating with hesitancy and pain on urination. Currently, his chronic conditions are well managed with metoprolol, lisinopril, and aspirin. Vital signs are temperature 100.2 F, blood pressure 135/75, pulse 76/min, and respiratory rate 15/min. Given this history, which of the following physical exam maneuvers are most helpful in making the diagnosis? Select all that apply. A. Abdominal palpation B. Costovertebral angle percussion C. Straight leg test D. Digital rectal exam E. Pinprick sensation of the legs

The correct answers are B and D. In an older male patient, prostatitis may present with low back pain. This patient's symptoms-general malaise, chills, hesitancy and pain on urination-and signs (fever) suggest acute bacterial prostatitis. Patients with acute bacterial prostatitis will often have exquisite tenderness over the prostate on rectal exam. This patient could also have pyelonephritis, which often goes along with costovertebral angle tenderness.


Conjuntos de estudio relacionados

CPR/First-Aid/Choking Victim Study Guide

View Set

Chapter 3 Quiz: The English Come to Stay 1600-1660

View Set

Lecture 7: Hormones/Endocrine System

View Set

Year 7 Maths - Equivalent fractions / Fractions of a Quantity / Writing one Number as A Fraction of Another

View Set

Defensive Driving Course: Final Exam

View Set

Quiz de culture générale sur la France

View Set